Q.1 An electronic device when fed with the numbers, rearranges them in a particular order following certain rules. The following is a step-by-step process of rearrangement for the given input of numbers.
Input :- 85 16 36 04 19 97 63 09
Step I :- 97 85 16 36 04 19 63 09
Step II :- 97 85 63 16 36 04 19 09
Step III :- 97 85 63 36 16 04 19 09
Step IV :- 97 85 63 36 19 16 04 09
Step V :- 97 85 63 36 19 16 09 04
(for the given input step V is the last step).
Which of the following will be the last step for the given input ?
Input :- 03 31 43 22 11 09
A) IV
B) V
C) VI
D) None of these
Ans. B
Clearly, in the given arrangement, the numbers have been arranged in descending order in a sequence, altering the position of only one number in each step
Input :- 03 31 43 22 11 09
Step I :- 43 03 31 22 11 09
Step II :- 43 31 03 22 11 09
Step III :- 43 31 22 03 11 09
Step IV :- 43 31 22 11 03 09
Step V :- 43 31 22 11 09 03
Q.2 Find the minimum number of straight lines required to make the given figure.
A)13
B)15
C)17
D)19
Ans. A
The figure may be labelled as shown.
The horizontal lines are IJ, AB, EF, MN, HG, DC and LK i.e. 7 in number.
The vertical lines are AD, EH, IL, FG, BC and JK i.e. 6 in number.
Thus, there are 7 + 6 = 13 straight lines in the figure.
Q.3 Priti Scored more than Rahul. Yamuna Scored as much as Divya. Lokita Scored less than manju. Rahul Scored more than Yamuna. Manju Scored less than Divya. Who Scored the lowest
A) Lokita
B) Manju
C) Yamuna
D) Priti
Ans. A
Priti>Rahul>Divya>Yamuna>Manju>Lokita
Q.4 Statements: All buildings are windows. No toys is building. Some tigers are toys.
Conclusions:
1) Some tigers are buildings.
2) Some windows are tigers.
3) All toys are tigers.
4) Some windows are toys.
Codes:
A) 1 & 3
B) 2 & 3
C) 1 & 2
D) None of the above
Ans. D
No toy is building. All buildings are windows.
Since the middle term ‘buildings’ is distributed twice and one premise is negative, the conclusion must be particular negative and should not contain the middle term.
So, it follows that ‘Some windows are not toys’.
Some tigers are toys. No toy is building.
Since one premise is particular and the other premise is negative, the conclusion must be particular negative and should not contain the middle term. So, it follows that ‘Some tigers are not buildings’.
Q.5 China wants to avoid financial collapse of their economy. In order to do this, China must raise their gross national product rate by 33 percent. China’s economy is structured so that if the 33 percent increase in GNP is reached, then it is possible for a 50 percent GNP increase.
Of the following statements listed below, which one must be true if is it to be believed that the above statements are also true?
A) If China’s 50 percent increase in GNP is unattainable, then its economy will collapse.
B) China’s GNP will not have a 50 percent increase if its economy falls.
C) The economy of China will not fall if it can obtain an increased GNP of 50 percent.
D) A 17 percent GNP increase will be unattainable if China continues to suffer national conflict.
Ans. A
The passage states China must raise its GNP by 33% to avoid economic collapse; if 33% is reached, 50% is possible. If 50% is impossible, 33% was not reached; the economy will collapse. China’s economy will fall without GNP increase, not vice versa (B); and by 33%, not 50% (B), (C). National conflict is not mentioned; neither is 17% (D) or 71% GNP increase
Passage
A charitable foundation awards grants in exactly four areas — medical services, theater arts, wildlife preservation, and youth services — each grant being in one of these areas. One or more grants are awarded in each of the four quarters of a calendar year. Additionally, over the course of a calendar year, the following must obtain:
1) Grants are awarded in all four areas.
2) No more than six grants are awarded.
3) No grants in the same area are awarded in the same quarter or in consecutive quarters.
4) Exactly two medical services grants are awarded.
5) A wildlife preservation grant is awarded in the second quarter.
Q.6 If a wildlife preservation grant and a youth services grant are awarded in the same quarter of a particular calendar year, then any of the following could be true that year EXCEPT:
A) A medical services grant is awarded in the second quarter.
B) A theater arts grant is awarded in the first quarter.
C) A theater arts grant is awarded in the second quarter.
D) A wildlife preservation grant is awarded in the fourth quarter.
Ans. A
Explanation
This question deals with the awarding of grants during the quarters of a calendar year. As an aid in visualizing this problem, we can set up a simple table with columns representing the four quarters. Since the fifth condition in the passage states that “[a] wildlife preservation grant is awarded in the second quarter,” we know that all possible solutions for any question based on the passage must include a wildlife preservation grant awarded in the second quarter, which we can represent like this:
1 2 3 4
W
The particular question here begins with the added supposition that “a wildlife preservation grant and a youth services grant are awarded in the same quarter of a particular calendar year.” One possible way this could be satisfied is to have a youth services grant awarded in the second quarter in addition to the wildlife grant awarded in that quarter:
1 2 3 4
W
Y
Another possibility would be to have a wildlife preservation grant and a youth services grant both being awarded in some quarter other than the second quarter. Given the condition that “[n]o grants in the same area are awarded in the same quarter or in consecutive quarters,” the only quarter in which a wildlife preservation grant could be awarded in addition to the second quarter is the fourth quarter. So the only alternative way to satisfy the added supposition is if both a wildlife preservation grant and a youth services grant are awarded in the fourth quarter:
1 2 3 4
W W
Y
So far, then, we’ve determined that for this question there must be a youth services grant awarded in the second quarter or the fourth quarter.
Each of the incorrect answer choices for this question is a statement that could be true. The question asks you to identify the exception; that is, you need to find the statement that cannot be true.We saw above that a youth services grant must either be awarded in the second quarter or the fourth quarter. On either possibility, awarding a youth services grant in the third quarter would result in two consecutive quarters where the youth services grant is awarded:
1 2 3 4
W
YY
or:
1 2 3 4
W Y W
Y
In both cases, two youth services grants would be awarded in consecutive quarters, in violation of the third condition.
To see that each of the other answer choices could be true, it will suffice to construct a possible outcome for each one that is consistent with the supposition given in the question and the conditions in the passage. Consider the following possible outcome:
1 2 3 4
T M
W
Y T M
A quick check of the conditions shows that this satisfies all of the conditions for the problem:
A wildlife preservation grant and a youth services grant are awarded in the same quarter of a particular calendar year.
Grants are awarded in all four areas. (The table includes at least one of each of the four letters — M, T, W, and Y.)
No more than six grants are awarded. (The table contains exactly six entries.)
No grants in the same area are awarded in the same quarter or in consecutive quarters. (In the table above, only T and M are repeated, and neither repetition appears in the same or consecutive columns.)
Exactly two medical services grants are awarded. (The table contains exactly two M’s, in columns 2 and 4.)
A wildlife preservation grant is awarded in the second quarter.
Notice that in this possible outcome, a medical services grant is awarded in the second quarter (answer choice (A)) and a theater arts grant is awarded in the first quarter (answer choice (B)). So answer choices (A) and (B) are both incorrect.
Now consider the following possible outcome:
1 2 3 4
M T
W M W
Y
A check of the conditions shows that this satisfies the supposition and all of the conditions. In this outcome, a theater arts grant is awarded in the second quarter (answer choice (C)) and a wildlife preservation grant is awarded in the fourth quarter (answer choice (D)). So answer choices (C) and (D) are also incorrect.
The most commonly selected incorrect answer choice for this question was response (A).
Passage
From a group of seven people — J, K, L, M, N, P, and Q — exactly four will be selected to attend a diplomat’s retirement dinner. Selection conforms to the following conditions:
1) Either J or K must be selected, but J and K cannot both be selected.
2) Either N or P must be selected, but N and P cannot both be selected.
3) N cannot be selected unless L is selected.
4) Q cannot be selected unless K is selected.
Q.7 If P is not selected to attend the retirement dinner, then exactly how many different groups of four are there each of which would be an acceptable selection?
A) one
B) two
C) three
D) four
Ans. C
Explanation
This question adds a new supposition to the original set of conditions — “P is not selected to attend the retirement dinner.” The task is to determine all of the different possible selections that are compatible with this new supposition. A compatible solution is one that violates neither the new supposition nor the original conditions.
Since the second condition states “either N or P must be selected…,” we can infer from the new supposition (P is not selected) and the second condition (either N or P, but not both, is selected) that N is selected. And since N is selected, we know from the third condition that L is selected. In other words every acceptable selection must include both L and N.
We are now in a good position to enumerate the groups of four which would be acceptable selections. The first condition specifies that either J or K, but not both, must be selected. So you need to consider the case where J (but not K) is selected and the case in which K (but not J) is selected. Let’s first consider the case where J (but not K) is selected. In this case, Q is not selected, since the fourth condition tells you that if K is not selected, then Q cannot be selected either. Since exactly four people must be selected, and since P, K, and Q are not selected, M, the only remaining person, must be selected. Since M’s selection does not violate any of the conditions or the new supposition, N, L, J, and M is an acceptable selection; in fact, it is the only acceptable selection when K is not selected. So far we have one acceptable selection, but we must now examine what holds in the case where K is selected.
Suppose that K is selected. In this case J is not selected, but Q may or may not be selected. If Q is selected, it is part of an acceptable selection — N, L, K, and
If Q is not selected, remembering that J and P are also not selected, M must be selected. This gives us our final acceptable selection — N, L, K, and M.
Thus there are exactly three different groups of four which make up acceptable selections, and (C) is the correct response.
Q.8 Looking at a portrait of a man, Harsh said, “His mother is the wife of my father’s son and I have no brothers and sisters”. At whose portrait was Harsh looking?
A) His son
B) His cousin
C) His uncle
D) His nephew
Ans. A
The person in the portrait is Harsh’s son.
Q.9 Observe the dots on a dice (one to six dots) in the following figures. How many dots are contained on the face opposite to that containing four dots?
A) 2
B) 3
C) 6
D) Can’t be determined
Ans. A
Q.10 Sara lives in a large city on the East Coast. Her younger cousin Marlee lives in the Mid-west in a small town with fewer than 1,000 residents. Marlee has visited Sara several times during the past five years. In the same period of time, Sara has visited Marlee only once.
A) Marlee likes Sara better than Sara likes Marlee.
B) Sara thinks small towns are boring.
C) Sara is older than Marlee.
D) Marlee wants to move to the East Coast.
Ans.C
Since the paragraph states that Marlee is the younger cousin, Sara must be older than Marlee. There is no information to support the other choices.
Q.11 Georgia is older than her cousin Marsha. Marsha’s brother Bart is older than Georgia. When Marsha and Bart are visiting with Georgia, all three like to play a game of Monopoly. Marsha wins more often than Georgia does.
A) When he plays Monopoly with Marsha and Georgia, Bart often loses.
B) Of the three, Georgia is the oldest.
C) Georgia hates to lose at Monopoly.
D) Of the three, Marsha is the youngest.
Ans. D
If Georgia is older than Marsha and Bart is older than Georgia, then Marsha has to be the youngest of the three. Choice b is clearly wrong because Bart is the oldest. There is no information in the paragraph to support either choice a or choice c.